Which one of the following, if true, would most weaken one of the author's arguments in the last paragraph?

jingjingxiao11111@gmail.com on April 11, 2020

Why is A wrong?

Why is A wrong? Thanks

Replies
Create a free account to read and take part in forum discussions.

Already have an account? log in

jordanbirnholtz on April 12, 2020

Hey, I'm writing to echo the above commenter. It seems like A would be a better fit than anything, because it says that the diversity requirement of these communities is actually satisfied. The next best option seems to offer evidence that it might be satisfied in the future. So why were we wrong?

Juan.23 on April 9, 2022

@Jing Jing & @Jordan,

From what I can tell, A is wrong because it states that people online are more accepting, but that doesn't mean that they are diverse! But B mentions directly weakens the claim that computers are unattainable to the people who can't afford them.

Emil-Kunkin on May 9, 2022

Hi Juan23,

Your explanation is spot on! The author notes that genuine diversity is needed for a community, and simply accepting diversity is not the same thing as actually being diverse.